Please use an equation step by step and please show the numbers do not use words

Answers

Answer 1

Answer:

What's the question????

Step-by-step explanation:


Related Questions

NO LINKS!!! Draw and solve each triangle.

1. a= 3, b= 4, C= 40°
2. a= 3, c= 2, B=110°
3. a= 12, b= 13, , c= 5
4. a= 5, b= 8, c = 9​

Answers

Answers:a = 3, b = 4, c = 2.5720;  A = 48.5702, B = 91.4298, C = 40a = 3, b = 4.1357, c = 2;  A = 42.9719, B = 110, C = 27.0281a = 12, b = 13, c = 5;  A = 67.3801, B = 90, C = 22.6199a = 5, b = 8, c = 9;  A = 33.5573, B = 62.1819, C = 84.2608

The missing values are highlighted in bold, which are approximate to four decimal places (with the exception of B = 90 in problem 3).

Lowercase letters a,b,c are side lengths; Uppercase letters A,B,C are angles opposite the sides mentioned.

The drawings are shown below.

==========================================================

Explanation:

Problem 1

Use the Law of Cosines to solve for side c

c^2 = a^2 + b^2 - 2*a*b*cos(C)

c^2 = 3^2 + 4^2 - 2*3*4*cos(40)

c^2 = 6.61493337

c = sqrt(6.61493337)

c = 2.57195128

c = 2.5720

Now apply the Law of Sines to find angle A

sin(A)/a = sin(C)/c

sin(A)/3 = sin(40)/2.57195128

sin(A) = 3*sin(40)/2.57195128

sin(A) = 0.74976647

A = arcsin(0.74976647)

A = 48.57015285

A = 48.5702

Once we know two angles of a triangle, we can then subtract their sum from 180 to get the third missing angle.

A+B+C = 180

B = 180 - (A+C)

B = 180 - (48.57015285+40)

B = 91.42984715

B = 91.4298

-----------------------------------

Problem 2

Like with the previous problem, we can use the Law of Cosines to solve for the missing side.

b^2 = a^2 + c^2 - 2*a*c*cos(B)

b^2 = 3^2 + 2^2 - 2*3*2*cos(110)

b^2 = 17.10424172

b = sqrt(17.10424172)

b = 4.13572747

b = 4.1357

Now use the Law of Sines to find angle A

sin(A)/a = sin(B)/b

sin(A)/3 = sin(110)/4.13572747

sin(A) = 3*sin(110)/4.13572747

sin(A) = 0.68164014

A = arcsin(0.68164014)

A = 42.9719427

A = 42.9719

Then we can wrap things up like so

A+B+C = 180

C = 180 - (A+B)

C = 180 - (42.9719427+110)

C = 27.0280573

C = 27.0281

-----------------------------------

Problem 3

Use the Law of Cosines to find angle A.

a^2 = b^2 + c^2 - 2*b*c*cos(A)

12^2 = 13^2 + 5^2 - 2*13*5*cos(A)

144 = 194 - 130*cos(A)

-130*cos(A) = 144 - 194

-130*cos(A) = -50

cos(A) = (-50)/(- 130)

cos(A) = 0.38461538

A = arccos(0.38461538)

A = 67.38013505

A = 67.3801

We could use the Law of Cosines to find angle B, but I'll use the Law of Sines instead.

sin(A)/a = sin(B)/b

sin(B) = b*sin(A)/a

sin(B) = 13*sin(67.38013505)/12

sin(B) = 0.99999999998574

sin(B) = 1

B = arcsin(1)

B = 90

It turns out we have a 5-12-13 right triangle (though usually c is the longest side instead of b).

Now we could use the Law of Cosines to find angle C, but the steps shown below are much quicker in my opinion that don't involve any trig functions.

A+B+C = 180

C = 180 - (A+B)

C = 180 - (67.38013505 + 90)

C = 22.61986495

C = 22.6199

-----------------------------------

Problem 4

The steps will basically be identical to problem 3, but with different numbers of course.

First we need the Law of Cosines to find angle A.

a^2 = b^2 + c^2 - 2*b*c*cos(A)

5^2 = 8^2 + 9^2 - 2*8*9*cos(A)

25 = 145 - 144*cos(A)

-144*cos(A) = 25 - 145

-144*cos(A) = -120

cos(A) = (-120)/(-144)

cos(A) = 0.83333333

A = arccos(0.83333333)

A = 33.55730976

A = 33.5573

Now use the Law of Sines to find angle B.

sin(A)/a = sin(B)/b

sin(B) = b*sin(A)/a

sin(B) = 8*sin(33.55730976)/5

sin(B) = 0.88443328

B = arcsin(0.88443328)

B = 62.18186103

B = 62.1819

Lastly,

A+B+C = 180

C = 180 - (A+B)

C = 180 - (33.55730976 + 62.18186103)

C = 84.26082921

C = 84.2608

-----------------------------------

Side notes:

The phrasing "solve the triangle" means to find all three sides and all three angles.Each answer is approximate with the exception of B = 90 in problem 3.Intermediate calculations were done to eight decimal places. The final answers are to four decimal places. In the first two problems, we can prove only one triangle is possible due to the SAS triangle congruence theorem.For the last two problems, only one triangle is possible because of the SSS triangle congruence theorem.I used GeoGebra to make each triangle shown below.

Look at VSW and ZTSW in the image below.
Which of the following is the best description for this pair of angles?
O complementary
O supplementary
O acute
O straight

Answers

Answer:

Supplementary

Step-by-step explanation:

VSW and TSW combine and make 180. Complementary is when they make up to 90

B. Supplementary
Supplementary

What % of people surveyed
were 6-10 and 21-25 years old

Answers

Add total people:

5 + 20 + 40 + 30 + 20 = 115 people

Add people 6-10 to 21-25 = 20 + 20 = 40

Divide the two age ranges by total people:

40/115 = 0.34782609

Multiply by 100 to get percent:

0.34782609 x 100 = 34.782609%  Round off as needed

Solve for x .

A) 48

B)42

C)47

D) 44

Answers

Answer:

B. 42.

Step-by-step explanation:

x/35 = 30/25

25x = 30*35

x = (30*35)/25

  = 1050/25

  = 42.

 

Determine if true:
3x5+8=23

Answers

Answer:

True

Step-by-step explanation:

3x5=15

15+8=23

23=23

Answer:

True

Step-by-step explanation:

3 x 5 = 15

15 + 8 = 23

26. Let S = (-1,0,2,4,7). Find f(S) if
a) f(x)= 1.
b) f(x)=2x + 1.
c) f(x) = x/5
. d) f(x)=(x2 + 1)/3).

Answers

Answer:

d.f(x)=(×2 +1)/3).

Step-by-step explanation:

I think that was my answer

For a car moving at a constant speed, the distance traveled varies directly with the time spent driving. If such a car travels 210 miles in 7 hours, how many
miles does it travel in 4 hours?

Answers

Answer:

120 miles.

Step-by-step explanation:

Use long division to find how many miles the car travels in 1 hour then we multiply by 4

: (7÷210=30)×4=120miles. Hope this helped ;D

zearn lesson really hard

Answers

Answer:7541

Step-by-step explanation:

all you have to do is add the bottom

1 All sides are congruent. Paralelogram Rhombus Rectangle Square​

Answers

Answer:

Rhombus

Square

Step-by-step explanation:

Rhombuses and squares have sides that are always congruent.

if 9 is added to a number x the result is greater than 17. Find the range of values of x

Answers

Answer:

See below.

Step-by-step explanation:

x+9>17

  -9  -9

x>8

-hope it helps

select the correct form for 562,932 I want the correct form​

Answers

Answer: five hundred sixty-two thousand nine hundred thirty-two

Step-by-step explanation: Student instructed to give the word form of a given value.

Answer:

The correct word form is:

Five hundred sixty-two thousand nine hundred thirty-two.

---

Thank you so much for participate in the community of Brainly.com

I hope help you :)

Kind regards, Antonio.

Pls help me solve part a and part b

Answers

I don’t know the answer to part A but Part B’s answer is D

The train station clock runs too fast and gains 5 minutes every 3 days.
How many minutes and seconds will it have gained at the end of 4 days?

Answers

Answer:

first find the average per day. 5/3= 1 & 2/3... 2/3 of a minute is 40 is 60/3=20 seconds. so 4 days would be the full 5 minutes plus another 1 minute and another 40 seconds.. so by day 4 it would be 6 min and 40 seconds.

3 ounces cost $273
Find the unit rate

Answers

Answer:

$91 per ounce

Step-by-step explanation:

If 3 ounces cost $273, we can divide both sides by 3 to find the unit rate. To make this clearer, we use the variable [tex]x[/tex] for ounces. Then, we have the equation

[tex]3x=273[/tex].

Dividing both sides by 3, we have

[tex]x=91[/tex].

The cost of one ounce is $91.

In a survey of a community, 58% like to listen to the radio only and 18% like to watch the T.V. only. If the number of people who like to listen to the radio is thrice the number of people who like to watch the T.V., then,
(i) Find percentage of people who donot like both of them.
ii Represent the above information in venn diagram. ​

Answers

The percentage of those that did not like both of them is 76%

Venn diagrams and set notation

Let the total number of people in the community be 100%

n(u) = 100

If 58% like to listen to the radio only and 18% like to watch the T.V. only, then;

n(R) = 58 - x

n(T) = 118 - x

where x is the percentage of people who do not like both of them. Taking the sum;

58 + x + 18 = 100

x + 76 = 100

x = 100 - 76

x = 24

Hence 24% of people like both of them.

The percentage of those that did not like both of them is 76%

Learn more on sets here: https://brainly.com/question/13458417

pls help me find the awnser​

Answers

Answer:

I have no clue thats why is got this app

Answer:

57 units³

Step-by-step explanation:

Volume = length • width • height

Fill in the numbers:

V = 3 4/5 • 2 1/2 • 6

I am making them into improper fractions to make it easier:

V = 19/5 • 5/2 • 6

Multiplying the length and the width:

V = 95/10 • 6

Multiplying the height in:

V = 57

NO LINKS!! A quarterback tosses a football to a receiver 40 yards downfield. The height of the football, f(x), in feet, can be modeled by f(x)= -0.0.25x+x+6, where x is the ball's horizontal distance, in yards, from the quarterback. What is the ball's maximum height and how far from the quarterback does this occur?​

Answers

Answer:

Maximum height = 16 yards

This occurs 20 yards from the quarterback.

Step-by-step explanation:

Given function:  [tex]f(x)=-0.025x^2+x+6[/tex]

To find the maximum (turning point), differentiate the function:

[tex]f('x)=-0.05x+1[/tex]

Set the derivative to zero and solve for [tex]x[/tex]:

[tex]\implies f'(x)=0[/tex]

[tex]\implies -0.05x+1=0[/tex]

[tex]\implies 0.05x=1[/tex]

[tex]\implies x=20[/tex]

Substitute found value of [tex]x[/tex] into the function to find the maximum height:

[tex]f(20)=-0.025(20)^2+(20)+6=16[/tex]

Therefore, the maximum height is 16 yards.

This occurs 20 yards from the quarterback.

The maximum height of the football can be 16 yards and this occurs 20 yards from the quarterback. The concepts of differentiation are used to solve this problem.

Differentiation is a fundamental mathematical concept that is generally used to obtain the rate of change or slopes of a line.

It is given that the height of the football is modeled as the following function:

f(x) = -0.025x² + x + 6

where x is the ball's horizontal distance (in yards) from the quarterback.

To obtain the maximum height of the ball, the equation is needed to be differentiated and set equal to zero as follows:

[tex]\dfrac{d}{dx}f(x) = 0\\\dfrac{d}{dx}(-0.025x^2+x+6)=0\\-0.025\times2x +1 = 0\\-0.05x+1 = 0\\x = \dfrac{1}{0.05}\\x = 20[/tex]

Substitute this value of x in f(x) to find the maximum height,

f(20) = -0.025(20)² + 20 + 6

f(20) = -0.025 × 400 + 26

f(20) = -10 + 26

f(20) = 16

Therefore, the maximum height is 16 yards and this occurs 20 yards from the quarterback.

Learn more about Differentiation here:

https://brainly.com/question/33433874

#SPJ3

The complete question is as follows:

A quarterback tosses a football to a receiver 40 yards downfield. The height of the football, f(x), in feet, can be modeled by f(x) = -0.025x²+x+6, where x is the ball's horizontal distance, in yards, from the quarterback. What is the ball's maximum height and how far from the quarterback does this occur?​

What is the center and radius for the circle with equation 2x2 - 8x + 2y2 +12y + 14 = 0?
A A
(2,-3); r = 6
B
00
(293); r = 16
(-2,3); r = 6
0
D
(-2,3); r = 16

Answers

Answer:

[tex](2,\, -3)[/tex].

[tex]r = \sqrt{6}[/tex].

Step-by-step explanation:

Let [tex](a,\, b)[/tex] denote the center of this circle. Let [tex]r[/tex] ([tex]r > 0[/tex]) denote the radius of this circle. The equation of this circle would be:

[tex](x - a)^{2} + (y - b)^{2} = r^{2}[/tex].

Expand to obtain an equivalent equation:

[tex]x^{2} + (- 2\, a) + y^{2} + (- 2\, b) + (a^{2} + b^{2} - r^{2}) = 0[/tex].

Since this equation and the given equation describe the same circle, their corresponding coefficients should match.

Notice that the coefficients of [tex]x^{2}[/tex] and [tex]y^{2}[/tex] in this equation are both [tex]1[/tex]. However, the corresponding coefficients in the given equation are both [tex]2[/tex]. Thus, divide both sides of the given equation by [tex]2\![/tex] to match the coefficients of the [tex]x^{2}[/tex] and [tex]y^{2}[/tex] terms:

[tex]x^{2} + (- 4\, x) + y^{2} + 6\, y + 7 = 0[/tex].

Coefficients of [tex]x^{2}[/tex] and [tex]y^{2}[/tex] are now matched after the division.Coefficients of [tex]x[/tex] should match: [tex](-4) = (-2\, a)[/tex], such that [tex]a = 2[/tex].Coefficients of [tex]y[/tex] should match: [tex]6 = (-2\, b)[/tex], such that [tex]b = (-3)[/tex].

The constants should also match. Thus:

[tex]a^{2} + b^{2} - r^{2} = 7[/tex].

Substitute in [tex]a = 2[/tex] and [tex]b = (-3)[/tex]; given that [tex]r > 0[/tex], the value of [tex]r[/tex] would be:

[tex]r = \sqrt{2^{2} + 3^{2} - 7} = \sqrt{6}[/tex].

6. How do you write 100 as a power of 10?

Answers

Answer:10 to the 2nd power

Step-by-step explanation:


Diane, Henry, and Abdul have a total of $82 in their wallets. Diane has $8 less than Henry. Abdul has 3 times what Henry has. How much does each

Answers

Answer:

let's suppose

Diane: D= H – 8

Henry: H

Abdul: A = 3H

D+H+A=82

D=10

H=18

A=54

for function f(x) =-3x+3 find f(0).

Answers

Answer:

f(0)=3

Step-by-step explanation:

f(0) = -3 x 0 + 3

f(0)=3

this season the baseball team won 3/5 of its games. assuming there’s no ties, what is the ratio of games won to games lost

Answers

Answer:

3:2

Step-by-step explanation:

if they won 3/5 games, then the other 2 were lost so the ratio would be 3:2

A bag of marbles contains five red marbles three green marbles and six yellow marbles what is the probability of randomly choosing a marble that is green

Answers

That should answer ur question!

how do you calculate Ending Inventory if beginning inventory is 15000 the purchases is 24,000 and the cost of good sold is 23,000.

Answers


The ending inventory is: $14000

Find the missing angle measure of the triangle.

Answers

Answer:

58

Step-by-step explanation:

65 + 57 = 122

A triangle is 180 degrees

180 - 122 = 58

58  is the missing measurement

I've tried this multiple times and cant seem to get the right answer could someone help solve this?

Answers

Answer:

[tex]\dfrac{6\pi -9\sqrt{3} }{16}[/tex]

Step-by-step explanation:

You need to rewrite [tex]\cos^2 \theta[/tex] using cos double angle identity:

[tex]\cos 2\theta= \cos^2\theta-\sin^2\theta[/tex]

[tex]\implies \cos 2\theta= \cos^2\theta-(1-\cos^2\theta)[/tex]

[tex]\implies \cos 2\theta=2 \cos^2\theta-1[/tex]

[tex]\implies \cos^2\theta=\dfrac12(\cos 2\theta+1)[/tex]

Then substitute this into the integration.

Please see the attachment for the full integration (it was clearer for me to type this in MS word than use the equation editor here)

Recall the double angle identity for cosines

[tex]{\boxed{\bf{2\cos^{2}(\theta)=1+\cos (2\theta)}}}[/tex]

Using this the integral becomes :

[tex]{:\implies \quad \displaystyle \sf \dfrac{1}{2}\int_{\tiny \dfrac{\pi}{3}}^{\tiny \dfrac{\pi}{2}}\{3\cos (\theta)\}^{2}d\theta}[/tex]

[tex]{:\implies \quad \displaystyle \sf \dfrac{1}{2}\int_{\tiny \dfrac{\pi}{3}}^{\tiny \dfrac{\pi}{2}}9\cos^{2}(\theta)d\theta}[/tex]

As we can take constant out of the integrand so we have

[tex]{:\implies \quad \displaystyle \sf \dfrac{9}{2}\int_{\tiny \dfrac{\pi}{3}}^{\tiny \dfrac{\pi}{2}}\cos^{2}(\theta)d\theta}[/tex]

Using the double angle identity for cosines ;

[tex]{:\implies \quad \displaystyle \sf \dfrac{9}{2}\int_{\tiny \dfrac{\pi}{3}}^{\tiny \dfrac{\pi}{2}}\bigg\{\dfrac{1+\cos (2\theta)}{2}\bigg\}d\theta}[/tex]

[tex]{:\implies \quad \displaystyle \sf \dfrac{9}{4}\int_{\tiny \dfrac{\pi}{3}}^{\tiny \dfrac{\pi}{2}}\{1+\cos (2\theta)\}d\theta}[/tex]

Now , as integrals follow distributive property, so we now have

[tex]{:\implies \quad \displaystyle \sf \dfrac{9}{4}\bigg(\int_{\tiny \dfrac{\pi}{3}}^{\tiny \dfrac{\pi}{2}}d\theta+\int_{\tiny \dfrac{\pi}{3}}^{\tiny \dfrac{\pi}{2}}\cos (2\theta)d\theta \bigg)}[/tex]

[tex]{:\implies \quad \displaystyle \sf \dfrac{9}{4}\bigg\{\bigg(\theta +\dfrac{\sin (2\theta)}{2}\bigg) \bigg|_{\tiny \dfrac{\pi}{3}}^{\tiny \dfrac{\pi}{2}}\bigg\}}[/tex]

[tex]{:\implies \quad \displaystyle \sf \dfrac{9}{4}\bigg[\dfrac{\pi}{2}-\dfrac{\pi}{3}+\dfrac{1}{2}\bigg\{\sin (\pi)-\sin \left(\dfrac{2\pi}{3}\right)\bigg\}\bigg]}[/tex]

[tex]{:\implies \quad \displaystyle \sf \dfrac{9}{4}\bigg\{\dfrac{3\pi -2\pi}{6}+\dfrac{1}{2}\bigg(-\dfrac{\sqrt{3}}{2}\bigg)\bigg\}}[/tex]

[tex]{:\implies \quad \displaystyle \sf \dfrac{9}{4}\bigg(\dfrac{\pi}{6}-\dfrac{\sqrt{3}}{4}\bigg)}[/tex]

[tex]{:\implies \quad \displaystyle \sf \dfrac{9}{4}\bigg(\dfrac{2\pi -3\sqrt{3}}{12}\bigg)}[/tex]

[tex]{:\implies \quad \displaystyle \sf \dfrac{3(2\pi -3\sqrt{3})}{4\times 4}}[/tex]

[tex]{:\implies \quad \displaystyle \sf \dfrac{6\pi -9\sqrt{3}}{16}}[/tex]

[tex]{:\implies \quad \displaystyle \bf \therefore \quad \underline{\underline{\int_{\tiny \dfrac{\pi}{3}}^{\tiny \dfrac{\pi}{2}}\{3\cos (\theta)\}^{2}d\theta =\dfrac{6\pi -9\sqrt{3}}{16}}}}[/tex]

Used Concepts :-

[tex]{\boxed{\displaystyle \bf \int dx=x+C}}[/tex]

[tex]{\boxed{\displaystyle \bf \int \cos (nx)dx=\dfrac{\sin (nx)}{n}+C}}[/tex]

Find the radius of the cylinder with the given volume
WILL GIVE BRAINLIEST

Answers

Answer:

6.71 mm

Step-by-step explanation:

Find radius R

Volume = πR²H

4500π = πR²*100

R² = 45

so R = √45 = 3√5 = 6.71 mm

Choose the angles that are coterminal with -225° with -45° -125° -225° -495° 45° 135° 225° 495°

Answers

Answer:

  -225°, 135°, 495°

Step-by-step explanation:

Adding or subtracting multiples of 360° will give angle values that are coterminal with a given angle.

  -225° -360° = -595° (not on the list)

  -225° +0·360° = -225°

  -225° +360° = 135°

  -225° +2·360° = 495°

Angles -225°, 135°, 495° are coterminal with -225°.

Numbers x and y both squared and added

And ASAP pls

Nira here :)​

Answers

Answer:

2x² + 4y²

according to your given information.......

2 ft is equal to how many meters?

Answers

Answer:

2 feet is equal to 0.609 meters

One meter is equal to roughly 3.281 feet.

So two feet is less than one meter.

To find how many meters a certain amount of feet is, you divide by 3.281

2 ÷ 3.281 = 0.609

2 feet is equal to 0.609 meters

Answer:

0.61 meter

Step-by-step explanation:

Other Questions
1. What type of text type is presented above? Which of the following is NOT one of the main types ofwriting?A NarrativeB InformationalC FigurativeD Opinion What is the sixteenth letter of the greek alphabet?. What is the difference between reflective and refractive telescope? What is the surface area? 1 in 2 in 2 in square inches Find a . b if a = 3i + 8j and b= -0.5i - 4j. Im having trouble understanding this can someone help please and thank you Chloe has 12 commemorative plates and 18 commemorative spoons. She wants to display them in groups throughout her house, each with the same combination of plates and spoons, with none left over. What is the greatest number of groups Chloe can display? What idea does the speaker convey by saying that his head is bloody, but unbowed?Even though he has been hurt, he remains optimistic. Even though he has been hurt, he remains optimistic. He has been hurt, which has caused him to keep his head down. He has been hurt, which has caused him to keep his head down. He has been in a war, but he is not giving up. He has been in a war, but he is not giving up. He will continue to fight, even though he is hurt ABE CDE. Find the length of altitude NE. What is renters insurance?helllllllllllllllllllllp meeeeeeeeee CAN ANY DRAW AN WINNCOTT SCRIBBLE IN ABOUT JUST AN HOUR OR 2 helpp plss 10 points Cold air makes the process of evaporation go faster.A.falseB.true Jane made a poster that had a length: 1 2/3 of feet and a width: 5/6 of feet. What is the perimeter of her poster? 5(3406 - 867) + (1.9 x 6) = ? who returns from france to join the president's cabinet (Hamilton) consider the scene in The Great Gatsby in which Tom confronts Gatsby at the Plaza Hotel in Daisy's presence in what way do the Dynamics among the three characters shift in the same site specific evidence from the text to support your analysis How many moles of oxygen will occupy a volume of 435 mL at 6780 torr and 28 C? Please Help!! Brainiest!!